LSAT and Law School Admissions Forum

Get expert LSAT preparation and law school admissions advice from PowerScore Test Preparation.

User avatar
 Dave Killoran
PowerScore Staff
  • PowerScore Staff
  • Posts: 5852
  • Joined: Mar 25, 2011
|
#27293
Complete Question Explanation
(The complete setup for this game can be found here: lsat/viewtopic.php?t=11501)

The correct answer choice is (C)

If you understood the principle at work in question #5, this question should feel relatively comfortable.

If lockers 1 and 2 are single lockers assigned to boys, they must be assigned to M and P, not necessarily in that order. The shared lockers must then be assigned to lockers 3, 4, or 5. Because N and T are assigned shared lockers that cannot be adjacent lockers, they must be assigned to lockers 3 and 5, not necessarily in that order. R must then be assigned to locker 4. And, because F is already assigned to locker 3, J must then be assigned to locker 5, leading to the following scenario:
June 00_M12_game#1_L9_explanations_game#2_#6_diagram_1.png
Accordingly, answer choice (C) is correct.

Get the most out of your LSAT Prep Plus subscription.

Analyze and track your performance with our Testing and Analytics Package.